Σελίδα 1 από 1

Όριο ολοκληρωμάτων

Δημοσιεύτηκε: Παρ Φεβ 15, 2019 6:51 pm
από M.S.Vovos
Έστω η συνεχής συνάρτηση f:\left [ -1,1 \right ]\longrightarrow \mathbb{R}. Να αποδείξετε ότι:

\displaystyle{\lim_{n\rightarrow \infty }\frac{\displaystyle \int_{-1}^{1}f(x)\left ( 1-x^{2} \right )^{n}\textup{d}x}{\displaystyle \int_{-1}^{1}\left ( 1-x^{2} \right )^{n}\textup{d}x}=f(0)} Φιλικά,
Μάριος



Μέχρι 17/02/2019.

Re: Όριο ολοκληρωμάτων

Δημοσιεύτηκε: Κυρ Φεβ 17, 2019 2:55 pm
από sot arm
M.S.Vovos έγραψε:
Παρ Φεβ 15, 2019 6:51 pm
Έστω η συνεχής συνάρτηση f:\left [ -1,1 \right ]\longrightarrow \mathbb{R}. Να αποδείξετε ότι:

\displaystyle{\lim_{n\rightarrow \infty }\frac{\displaystyle \int_{-1}^{1}f(x)\left ( 1-x^{2} \right )^{n}\textup{d}x}{\displaystyle \int_{-1}^{1}\left ( 1-x^{2} \right )^{n}\textup{d}x}=f(0)} Φιλικά,
Μάριος



Μέχρι 17/02/2019.
Βάζω μια λύση παρακάτω με κάποια επιφύλαξη για τυχόν λογιστικό σφάλμα, έστω:

\displaystyle{L(f(x))=\lim_{n\rightarrow \infty }\frac{\displaystyle \int_{-1}^{1}f(x)\left ( 1-x^{2} \right )^{n}\textup{d}x}{\displaystyle \int_{-1}^{1}\left ( 1-x^{2} \right )^{n}\textup{d}x}}

Παρατηρώ ότι είναι γραμμική, πράγματι:

\displaystyle{L(cf+g)=cL(f)+L(g), c \in \mathbb{R}} , προκύπτει απλά με πράξεις, και:

\displaystyle{L(c)=\lim_{n\rightarrow \infty }\frac{\displaystyle \int_{-1}^{1}c\left ( 1-x^{2} \right )^{n}\textup{d}x}{\displaystyle \int_{-1}^{1}\left ( 1-x^{2} \right )^{n}\textup{d}x}=c \lim_{n\rightarrow \infty }\frac{\displaystyle \int_{-1}^{1}\left ( 1-x^{2} \right )^{n}\textup{d}x}{\displaystyle \int_{-1}^{1}\left ( 1-x^{2} \right )^{n}\textup{d}x}=c}

Εξετάζουμε τώρα μονώνυμα της μορφής x^{l} , αν l περιττός άμεσα το όριο μηδέν αφού η συνάρτηση εντός του ολοκληρώματος του αριθμητή είναι περιττή,και το ολοκλήρωμα είναι σε συμμετρικό γύρω από το 0 διάστημα.

Αν πάλι l άρτιος προκύπτει με παραγοντική:

\displaystyle{\int_{-1}^{1}x^{2k}(1-x^{2})^{n}dx=\frac{2n}{2k+1}\int_{-1}^{1}x^{2k+2}(1-x^{2})^{n}dx=...=\frac{2^{n+1}n!}{\prod_{i=0}^{n}(2k+2i+1)}}

Ο παρονομαστής είναι εφαρμογή του παραπάνω για κ=0, έχω λοιπόν για το κλάσμα:

\displaystyle{L(x^{2k})=\lim_{n\rightarrow +\infty}\frac{\prod_{i=0}^{n}(2i+1)}{\prod_{i=0}^{n}(2k+2i+1)}=\lim_{n\rightarrow +\infty}\frac{\prod_{i=1}^{n}(2i+1)}{\prod_{i=0}^{n}(2k+2i+1)}= 0}

Από τα παραπάνω για κάθε πολυώνυμο έχω:
\displaystyle{L(p(x))=p(0)}
αφού όλες οι δυνάμεις του x δίνουν μηδέν και ο σταθερός όρος ισούται με p(0)

Έστω ε>0 τυχόν, η f είναι συνεχής στο [-1,1] άρα για το δοθέν ε υπάρχει πολυώνυμο έτσι ώστε:

\displaystyle{|f(x)-p(x)|<\frac{\epsilon}{2}} (Weierstrass)

Άρα:
\displaystyle{|L(f(x)-f(0))|=|L(f(x)-f(0))-L(p(x)-p(0))|=|L(f(x)-p(x))+L(f(0)-p(0)))|\leq}

\displaystyle{|L(f(x)-p(x))|+|L(f(0)-p(0))|\leq L(|f(x)-p(x)|)+L(|f(0)-P(0)|) <\frac{\epsilon}{2}+\frac{\epsilon}{2}=\epsilon}}

και τελειώσαμε, ξαναλέω με κάποια επιφύλαξη για λογιστικό σφάλμα, αλλά νομίζω η ιδέα είναι η σωστή.

Re: Όριο ολοκληρωμάτων

Δημοσιεύτηκε: Κυρ Φεβ 17, 2019 6:42 pm
από ΠΑΠΑΔΟΠΟΥΛΟΣ ΣΤΑΥΡΟΣ
sot arm έγραψε:
Κυρ Φεβ 17, 2019 2:55 pm
M.S.Vovos έγραψε:
Παρ Φεβ 15, 2019 6:51 pm
Έστω η συνεχής συνάρτηση f:\left [ -1,1 \right ]\longrightarrow \mathbb{R}. Να αποδείξετε ότι:

\displaystyle{\lim_{n\rightarrow \infty }\frac{\displaystyle \int_{-1}^{1}f(x)\left ( 1-x^{2} \right )^{n}\textup{d}x}{\displaystyle \int_{-1}^{1}\left ( 1-x^{2} \right )^{n}\textup{d}x}=f(0)} Φιλικά,
Μάριος



Μέχρι 17/02/2019.
Βάζω μια λύση παρακάτω με κάποια επιφύλαξη για τυχόν λογιστικό σφάλμα, έστω:

\displaystyle{L(f(x))=\lim_{n\rightarrow \infty }\frac{\displaystyle \int_{-1}^{1}f(x)\left ( 1-x^{2} \right )^{n}\textup{d}x}{\displaystyle \int_{-1}^{1}\left ( 1-x^{2} \right )^{n}\textup{d}x}}

Παρατηρώ ότι είναι γραμμική, πράγματι:

\displaystyle{L(cf+g)=cL(f)+L(g), c \in \mathbb{R}} , προκύπτει απλά με πράξεις, και:

\displaystyle{L(c)=\lim_{n\rightarrow \infty }\frac{\displaystyle \int_{-1}^{1}c\left ( 1-x^{2} \right )^{n}\textup{d}x}{\displaystyle \int_{-1}^{1}\left ( 1-x^{2} \right )^{n}\textup{d}x}=c \lim_{n\rightarrow \infty }\frac{\displaystyle \int_{-1}^{1}\left ( 1-x^{2} \right )^{n}\textup{d}x}{\displaystyle \int_{-1}^{1}\left ( 1-x^{2} \right )^{n}\textup{d}x}=c}

Εξετάζουμε τώρα μονώνυμα της μορφής x^{l} , αν l περιττός άμεσα το όριο μηδέν αφού η συνάρτηση εντός του ολοκληρώματος του αριθμητή είναι περιττή,και το ολοκλήρωμα είναι σε συμμετρικό γύρω από το 0 διάστημα.

Αν πάλι l άρτιος προκύπτει με παραγοντική:

\displaystyle{\int_{-1}^{1}x^{2k}(1-x^{2})^{n}dx=\frac{2n}{2k+1}\int_{-1}^{1}x^{2k+2}(1-x^{2})^{n}dx=...=\frac{2^{n+1}n!}{\prod_{i=0}^{n}(2k+2i+1)}}

Ο παρονομαστής είναι εφαρμογή του παραπάνω για κ=0, έχω λοιπόν για το κλάσμα:

\displaystyle{L(x^{2k})=\lim_{n\rightarrow +\infty}\frac{\prod_{i=0}^{n}(2i+1)}{\prod_{i=0}^{n}(2k+2i+1)}=\lim_{n\rightarrow +\infty}\frac{\prod_{i=1}^{n}(2i+1)}{\prod_{i=0}^{n}(2k+2i+1)}= 0}

Από τα παραπάνω για κάθε πολυώνυμο έχω:
\displaystyle{L(p(x))=p(0)}
αφού όλες οι δυνάμεις του x δίνουν μηδέν και ο σταθερός όρος ισούται με p(0)

Έστω ε>0 τυχόν, η f είναι συνεχής στο [-1,1] άρα για το δοθέν ε υπάρχει πολυώνυμο έτσι ώστε:

\displaystyle{|f(x)-p(x)|<\frac{\epsilon}{2}} (Weierstrass)

Άρα:
\displaystyle{|L(f(x)-f(0))|=|L(f(x)-f(0))-L(p(x)-p(0))|=|L(f(x)-p(x))+L(f(0)-p(0)))|\leq}

\displaystyle{|L(f(x)-p(x))|+|L(f(0)-p(0))|\leq L(|f(x)-p(x)|)+L(|f(0)-P(0)|) <\frac{\epsilon}{2}+\frac{\epsilon}{2}=\epsilon}}

και τελειώσαμε, ξαναλέω με κάποια επιφύλαξη για λογιστικό σφάλμα, αλλά νομίζω η ιδέα είναι η σωστή.
Στην σχέση
\displaystyle{\int_{-1}^{1}x^{2k}(1-x^{2})^{n}dx=\frac{2n}{2k+1}\int_{-1}^{1}x^{2k+2}(1-x^{2})^{n}dx
υπάρχει τυπογραφικό.
Είναι
\displaystyle{\int_{-1}^{1}x^{2k}(1-x^{2})^{n}dx=\frac{2n}{2k+1}\int_{-1}^{1}x^{2k+2}(1-x^{2})^{n-1}dx

το τελικό δεν νομίζω ότι έχει κάποια σημασία.



Η υπόθεση της συνέχειας της f παντού είναι περιττή.

Το αποτέλεσμα ισχύει αν η f είναι συνεχής στο 0 και έχουμε ακόμα μια συνθήκη.
π.χ f φραγμένη η
το \displaystyle \int_{-1}^{1}|f(x)|dx υπάρχει.

Re: Όριο ολοκληρωμάτων

Δημοσιεύτηκε: Τρί Φεβ 19, 2019 12:38 am
από mikemoke
Έστω ότι f συνεχής για x=0 και φραγμένη.
s_n=\frac{\int_{-1}^{1}f(x)(1-x^2)^ndx}{\int_{-1}^{1}(1-x^2)^ndx}= \frac{\int_{-\delta }^{\delta }f(x)(1-x^2)^ndx}{\int_{-1}^{1}(1-x^2)^ndx}+\frac{\int_{A_{\delta}} f(x)(1-x^2)^ndx}{\int_{-1}^{1}(1-x^2)^ndx} (1)
όπου A_\delta =[-1,1]\setminus [-\delta ,\delta ] για \delta \in (0,1)
Ισχύεί ότι :
a)\frac{\int_{-\delta }^{\delta }(1-x^2)^ndx}{\int_{-1}^{1}(1-x^2)^n}\rightarrow 1
b)\frac{\int_{A_\delta }(1-x^2)^ndx}{\int_{-1}^{1}(1-x^2)^ndx}\rightarrow 0
Έστω \epsilon >0
τότε \exists M>0\forall x\in[-1,1]:|f(x)|\leq M
\exists \delta >0: f(0)-\epsilon /3<f(x)<f(0)+\epsilon /3
Από (1) έχουμε ότι
(f(0)-\epsilon /3)\frac{\int_{-\delta }^{\delta }(1-x^2)^ndx}{\int_{-1}^{1}(1-x^2)^ndx}+(-M)\frac{\int_{A_{\delta}} (1-x^2)^ndx}{\int_{-1}^{1}(1-x^2)^ndx} \leq s_n\leq (f(0)+\epsilon /3)\frac{\int_{-\delta }^{\delta }(1-x^2)^ndx}{\int_{-1}^{1}(1-x^2)^ndx}+M\frac{\int_{A_{\delta}} (1-x^2)^ndx}{\int_{-1}^{1}(1-x^2)^ndx}

Aπό a,b) έπεται ότι \exists n_0\in\mathbb{N}\forall n\geq n_0:f(0)-\epsilon <s_n<f(0)+\epsilon
Άρα s_n\rightarrow f(0)

Για a,b) βλέπε εδώ :https://math.stackexchange.com/question ... 18_3118095

Re: Όριο ολοκληρωμάτων

Δημοσιεύτηκε: Τρί Φεβ 19, 2019 12:55 am
από Λάμπρος Κατσάπας
Την άσκηση σε όλη της τη γενικότητα μπορεί κανείς να την βρει στον Spivak, Κεφάλαιο ''Παράγωγοι και Ολοκληρώματα'' ή στον Παπαδημητράκη άσκηση 7.3. 24. Φαντάζομαι και αλλού μιας και είναι all time classic.

Re: Όριο ολοκληρωμάτων

Δημοσιεύτηκε: Τρί Φεβ 19, 2019 1:50 am
από ΠΑΠΑΔΟΠΟΥΛΟΣ ΣΤΑΥΡΟΣ
Αν θέσουμε
\displaystyle p_{n}(x)=\frac{ ( 1-x^{2} )^{n}}{\int_{-1}^{1} ( 1-x^{2})^{n}dx}
θέλουμε να δείξουμε ότι
για κάθε
f:[-1,1]\rightarrow \mathbb{R}
συνεχή στο 0 και ολοκληρώσιμη
ισχύει
\displaystyle \int_{-1}^{1}f(x)p_{n}(x)dx\rightarrow f(0)

Για να το δείξουμε
αρκούν τρεις ιδιότητες που έχουν.

1)p_{n}(x)\geq 0,n=1,2,..

2)\int_{-1}^{1}p_{n}(x)dx=1,n=1,2,...

3)Για 0<\delta <1

είναι \lim_{n\rightarrow \infty }sup\left \{ |p_{n}(x)|:|x|> \delta \right \}=0

Η 1)και 2) είναι τετριμένες
Για την 3)
εχουμε
\displaystyle \int_{-1}^{1}(1-x^{2})^{n}dx\geq \int _{|x|\leq \frac{1}{\sqrt{n}}}(1-x^{2})^{n}dx\geq \frac{2}{\sqrt{n}}(1-\frac{1}{n})^{n}\geq C\frac{1}{\sqrt{n}}

οπότε

sup\left \{ |p_{n}(x)|:|x|> \delta \right \}\leq C\sqrt{n}(1-\delta ^{2})^{n}

που παίρνοντας όριο το έχουμε.

Η απόδειξη πάει ως εξης.(θα την γράψω σύντομα)

\displaystyle \int_{-1}^{1}f(x)p_{n}(x)dx- f(0)=\int_{-1}^{1}(f(x)-f(0))p_{n}(x)dx=\int _{|x|< \delta }+\int _{|x|\geq \delta }=I_{1}+I_{2}

Για το I_{1} είναι

\displaystyle |I_{1}|=|\int_{-\delta }^{\delta }(f(x)-f(0))p_{n}(x)dx|\leq \int_{-\delta }^{\delta }|f(x)-f(0)|p_{n}(x)dx
\displaystyle \leq \epsilon \int_{-\delta }^{\delta }p_{n}(x)dx\leq \epsilon \int_{-1 }^{1 }p_{n}(x)dx=\epsilon

Ενώ για το I_{2} έχουμε αν ονομάσουμε

Q_{n}=sup\left \{ |p_{n}(x)|:|x|> \delta \right \}

|I_{2}|\leq Q_{n}\int_{-1}^{1}|f(x)|dx

που προφανώς πάει στο 0

Φυσικά όλα τα παραπάνω είναι πασίγνωστα σε αυτούς που κάνουν κατανομές.
Στην ουσία η ακολουθία (p_{n})_{n\in \mathbb{N}}
συγκλίνει στην  \delta του Dirac.